Direkte Summe von Hilbert-Räumen

Ich bin ein Physiker, der sich nicht so gut mit mathematischer Strenge auskennt (schade, ich weiß! Aber ich arbeite daran.) In Walds Buch über QFT in gekrümmten Raumzeiten fand ich die folgenden Definitionen der direkten Summe von Hilbert Räume. Er sagt -

Als nächstes definieren wir die direkte Summe von Hilbert-Räumen. Lassen { H a } sei eine beliebige Sammlung von Hilbert-Räumen, indiziert durch a (Wir werden uns nur für den Fall interessieren, in dem es höchstens eine abzählbare Anzahl von Hilbert-Räumen gibt, aber für diese Konstruktion braucht keine solche Einschränkung gemacht zu werden). Die Elemente des kartesischen Produkts × a H a bestehen aus der Sammlung von Vektoren { Ψ a } für jede Ψ a H a . Betrachten Sie nun die Teilmenge v × a H a , bestehend aus Elementen, für die alle, aber endlich viele der Ψ a verschwinden. Dann v hat die natürliche Struktur eines inneren Produktraums. Wir definieren den Direktsummen-Hilbert-Raum a H a um die Hilbert-Raum-Vervollständigung zu sein v . Daraus folgt im Fall einer abzählbar unendlichen Sammlung von Hilbert-Räumen { H ich } jede Ψ ich H ich besteht aus beliebigen Folgen { Ψ ich } so dass jeder Ψ ich H ich Und ich Ψ ich ich 2 < .

Hier     ich ist die Norm definiert in H ich . Auch Hilbert-Raum-Vervollständigung eines inneren Produktvektorraums v ist ein Raum H so dass v H Und H in der zugehörigen Norm vollständig ist. Es ist konstruiert aus v indem Äquivalenzklassen von Cauchy-Folgen aufgenommen werden v .

Jetzt die Fragen -

1. Warum nicht v die Struktur eines inneren Produktraums haben?

2. Wie kommt er darauf? ich Ψ ich ich 2 < ?

3. Wie passt diese Definition der direkten Summe zu den üblichen Dingen, die wir sehen, wenn wir Tensoren in der allgemeinen Relativitätstheorie oder in Darstellungen von Lie-Algebren usw. betrachten?

PS - Ich habe auch ein ähnliches Problem mit Walds Definition eines Tensorprodukts von Hilbert-Räumen. Ich habe beschlossen, dies in einer separaten Frage zu behandeln. Wenn Sie diese Frage beantworten könnten, sollten Sie sich diese auch ansehen. Es ist hier zu finden . Danke!

Zu Tensoren siehe auch physical.stackexchange.com/q/32011/2451 und darin enthaltene Links.
Für diejenigen, die mehr "in mathematischer Strenge" versiert sind, ist eine direkte Summe in vielen Fällen ein "Nebenprodukt". Dies wird durch eine "universelle Eigenschaft" definiert. Ich finde sehr interessant, dass ein Objekt durch seine Eigenschaft definiert wird und nicht durch eine explizite Konstruktion. Soweit ich weiß, ist die allgemeine Einstellung für die Definition der direkten Summe die abelschen Kategorien.
Und was mich schon sehr lange beschäftigt, ist vgl. math.stackexchange.com/q/523670 . Zur letzten Frage vgl. math.stackexchange.com/q/646227
Ich habe eine andere Frage gefunden: Warum muss keine Einschränkung auf den zählbaren Indexsatz vorgenommen werden? héhé, es ist die übliche unzählbare Summe ich Ψ ich ich 2 das ist eigentlich ein zählbares Argument
@ user39158 Ist die direkte Summenkonstruktion ein Nebenprodukt für Hilbert Spaces?
Ja ist es. (Weitere irrelevante Details: Obwohl das, was ich über abelsche Kategorien gesagt habe, falsch ist: In einer der Antworten in math.stackexchange.com/questions/2001/… heißt es, dass Hilbert-Räume keine abelsche Kategorie bilden. Ich habe jedoch Dinge gesehen wie "Existenz von Nebenprodukten für jede endliche Familie von Objekten" äquivalent zu "Existenz von Colimits" oder so ähnlich.)

Antworten (3)

Die beiden anderen Antworten hier folgen Walds Ansatz nicht direkt, obwohl sie immer noch ungefähr richtig sind. Hier ist eine Antwort, die genau seinen Anweisungen folgt und die meisten Details zeigt. Aufgrund der Tatsache, dass Sie über QM sprechen, gehe ich davon aus, dass Sie komplexe Hilbert-Räume verwenden, obwohl es keinen wesentlichen Unterschied zu echten Hilbert-Räumen gibt.

In meiner Antwort habe ich eine gewisse Vertrautheit mit der realen Analyse angenommen, aber nicht mit der Funktionsanalyse (da dies eine grundlegende Konstruktion aus der Funktionsanalyse ist). Wenn Sie nicht wissen, was eine Cauchy-Folge in einem metrischen Raum ist, sollten Sie sich damit vertraut machen, bevor Sie mit Teil 2 beginnen. Außerdem hat diese Antwort wirklich keine Physik; es ist alles Funktionsanalyse (das Studium unendlich dimensionaler Vektorräume).

1. Warum nicht v die Struktur eines inneren Produktraums haben?

Er hat definiert v als Teilmenge von × a H a wo alle, aber endlich viele Ψ a verschwinden. Das heißt, ein generisches Element von v ist eine Auswahl von Ψ a für jedes Alpha, wo die einzigen Nicht-Null-Elemente sind Ψ a 1 , Ψ a 2 , , Ψ a N für eine positive ganze Zahl N und eine Auswahl an Indizes a 1 , , a N . Beachten Sie, dass verschiedene Elemente Ψ Und Ψ ' wird im Allgemeinen verschiedenen positiven ganzen Zahlen entsprechen N , N ' und verschiedene Indizes a 1 , , a N Und a 1 ' , , a N ' ' . Für den Rest dieses Abschnitts übernehme ich Ψ Und Ψ ' zwei generische Elemente von sein v durch die obigen Formeln gegeben.

Wir wollen ein inneres Produkt anziehen v . Normalerweise würden wir für endliche direkte Summen so etwas tun wollen Ψ | Ψ ' = a Ψ a | Ψ a ' a , Wo | a ist das innere Produkt auf H a . Aber wir haben eine unendliche Summe, die bedeutungslos ist, bis wir einen Weg finden, sie zu interpretieren.

Glücklicherweise ist dies nicht wirklich eine unendliche Summe, gerade wegen der Einschränkung, die wir gemacht haben. Alle bis auf endlich viele Terme sind 0. Für alle a Um einen Term ungleich Null beizutragen, muss er Mitglied beider Sätze sein a ich 's und der Satz von a J ' 'S. Lassen Sie uns diese Indizes mit benennen β , so dass { β k } = { a ich } { a J ' } . Jetzt können wir der obigen Summe einen Sinn geben, indem wir sie nur auf die Indizes beschränken, die Terme ungleich Null beitragen, so dass Ψ | Ψ ' = β k Ψ β k | Ψ β k ' β k . Sie könnten beides genauso gut zusammenfassen a ich oder a J ' Indizes, was natürlich immer noch eine endliche Summe wäre, aber ich ziehe es vor, es auf diese Weise offensichtlich (konjugiert) symmetrisch zu machen.

Sie müssen an dieser Stelle überprüfen, ob die Axiome für ein inneres Produkt durch diese Wahl von erfüllt sind | . Es ist eindeutig symmetrisch bis zu einem komplexen Konjugat, was wir für ein hermitisches inneres Produkt wollen. Die Tatsache, dass es linear ist, erfordert etwas Herumspielen mit Symbolen, wenn Sie beweisen wollen, obwohl die Richtung des Beweises sofort offensichtlich sein sollte, wenn Sie dem oben Gesagten gefolgt sind. Positive Bestimmtheit wird von der vererbt H a . Speziell, Ψ 2 = Ψ | Ψ = a ich Ψ a ich | Ψ a ich a ich = a ich Ψ a ich a ich 2 was eine endliche Summe von ist N positive Terme durch positive Bestimmtheit des Skalarprodukts auf jedem der direkten Summanden, also Ψ 2 > 0 (so lange wie N 0 , Aber N = 0 ist der Fall, wo Ψ = 0 was natürlich Norm hat 0 ).

Sie könnten sagen: "Ich möchte nur zählbare direkte Summen, kann ich also nicht einfach die Grenze von Teilsummen nehmen?". Sie können das durchaus tun, aber es funktioniert nicht für unzählige direkte Summen (die Sie möglicherweise nicht benötigen). Sie müssen auch nachweisen, dass die Art und Weise, wie Sie die bestellen a ändert das Endergebnis für kein inneres Produkt und alle möglichen anderen technischen Details. Am Ende sind diese technischen Details so ziemlich alles andere, was wir für den allgemeinen Fall sowieso tun müssen. Wenn Sie dies tun möchten, sollten Sie auch den Hinweis im nächsten Abschnitt befolgen.

2. Wie kommt er darauf? ich Ψ ich ich 2 < ?

Lassen Sie uns klarstellen, dass wir jetzt mit einer zählbaren direkten Summe arbeiten, sodass die Summanden als gewählt werden können H ich für ich = 1 , 2 , . Dies ist immer noch (möglicherweise) eine unendliche Sammlung, aber jetzt können wir ein allgemeines Element von schreiben × ich H ich als Folge Ψ = ( Ψ 1 , Ψ 2 , ) . Ψ v würde bedeuten, dass alle, aber endlich viele der Ψ ich Sind 0 , oder äquivalent, dass es einen Index gibt M so dass für alle ich > M , Ψ ich = 0 .

Lassen Sie uns auch klarstellen, was er hier sagt. Er behauptet, dass ein generisches Element von ich H ich , der Abschluss von v bezogen auf das innere Produkt | , ist eine beliebige Folge Ψ = ( Ψ 1 , Ψ 2 , ) so dass ich Ψ ich ich 2 < S. Ψ Hier ist im Allgemeinen kein Element von v , sondern seiner Vollendung als Hilbert-Raum.

Hinweis: Wenn Sie irgendein Teil des Folgenden verwirrt, wäre es kein schwerwiegender Fehler, an die Definition von zu denken ich H ich eine abzählbare direkte Summe von Hilbert-Räumen als Unterraum von × ich H ich bestehend aus all diesen Vektoren Ψ in letzterem, die befriedigen ich Ψ ich ich 2 < . So wird es in einigen Texten und Wikipedia hier definiert . Wiki hat sowohl diese Definition als auch Walds hier , wo es feststellt, dass sie ohne Beweis gleichwertig sind. Der vollständige Beweis, dass dies ein Hilbert-Raum ist, befindet sich auf dieser ProofWiki-Seite , obwohl einige davon im Wesentlichen unten dupliziert werden. Wichtig ist, dass wir wollen, dass das innere Produkt endlich und vollständig ist, sodass wir tatsächlich einen Hilbert-Raum erhalten, und diese Bedingung stellt dies sicher.


Ich werde die Fertigstellung von konstruieren v in diesem Abschnitt. Wenn Sie dies bereits verstehen oder aufgeben, es zu verstehen, können Sie gerne zum nächsten Abschnitt springen, wo ich zu Ihrer eigentlichen Frage komme. Dies ist meiner Meinung nach der schwierigste Teil der Antwort, und hier müssen Sie wirklich ein bisschen echte Analyse kennen. Allerdings gibt es hier keine Physik, also überspringen wir ein bisschen die technischen Einzelheiten.

Wir wollen abschließen v bezogen auf das innere Produkt | , oder genauer gesagt in Bezug auf die Metrik an v definiert durch das Skalarprodukt, das gegeben ist durch D ( Ψ , Ψ ' ) = Ψ Ψ ' = Ψ Ψ ' | Ψ Ψ ' . Wir müssen dies tun, weil wir immer noch einen Hilbert-Raum wollen und Hilbert-Raum mit vollständigen inneren Produkten geliefert wird. In endlichen Dimensionen ist jedes innere Produkt vollständig, aber in unendlichen Dimensionen ist das nicht wahr. Es gibt Cauchy-Folgen, von denen wir hoffen würden, dass sie konvergieren, aber das tun sie tatsächlich nicht. Lassen Sie uns zum Beispiel für einen Moment los H ich = C für jede ich . Dann der Vektor ( 1 , 1 / 2 , 1 / 3 , 1 / 4 , ) ist nicht drin v da es unendlich viele Nicht-Null-Einträge hat. Wir können jedoch eine Folge von Elementen von schreiben v die gegen diesen Vektor konvergieren "sollten". ( 1 , 0 , 0 , 0 ) , ( 1 , 1 / 2 , 0 , 0 , ) , ( 1 , 1 / 2 , 1 / 3 , 0 , 0 , ) , . Dies ist eine Cauchy-Folge, aber sie konvergiert nicht v . Also müssen wir etwas hinzufügen v um diese Folge (und andere ähnliche) konvergieren zu lassen, sodass wir einen Hilbert-Raum erhalten.

Der Abschluss von v folgt genau den gleichen Schritten wie die Vervollständigung eines beliebigen metrischen Raums in Bezug auf Äquivalenzklassen von Cauchy-Folgen. Der bekannteste Fall ist wahrscheinlich die Konstruktion der reellen Zahlen aus den rationalen Zahlen, was auch der pathologischste Fall ist, da wir uns normalerweise auf die Vollständigkeit der reellen Zahlen berufen müssen, was bei der Konstruktion von reellen Zahlen keine Option ist Zahlen zum ersten Mal. In diesem Fall gibt es jedoch keine solche Schwierigkeit. Wikipedia deckt dies recht gut ab.

Auf jeden Fall denke ich, dass es für den Spezialfall von Hilbert-Räumen am aufschlussreichsten (und am wenigsten offensichtlichen) ist, zu sehen, warum die Vervollständigung von v kann als Unterraum von angesehen werden × ich H ich . Ein Vektor in der Vervollständigung von v ist eine Äquivalenzklasse von Cauchy-Folgen. Lassen Sie uns eine bestimmte repräsentative Sequenz für diese Klasse auswählen. Um zu vermeiden, dass die Notation überladen wird, schreibe ich diese Cauchy-Folge hinein v als Ψ 1 , Ψ 2 , Ψ 3 , . Jedes Element Ψ J dieser Sequenz ist selbst ein Element von v , es handelt sich also um eine Folge des Formulars ( Ψ 1 J , Ψ 2 J , ) , von denen nur endlich viele Terme ungleich Null sind. Die Folge ist Cauchy, also für jeden ϵ > 0 , es gibt einige N so dass für jede Wahl von J , k > N , Ψ J Ψ k < ϵ . Das bedeutet insbesondere, dass für jede Wahl von ich , Ψ ich J Ψ ich k < ϵ , also für jeden ich , der Ablauf ( Ψ ich 1 , Ψ ich 2 , ) ist eine Cauchy-Folge in H ich .

Jetzt, H ich ist nach Annahme ein Hilbertraum. Das bedeutet, dass diese Cauchy-Folge ( Ψ ich 1 , Ψ ich 2 , ) konvergiert zu etwas in H ich . Nennen wir das etwas Ψ ich . Wir tun dies für jeden ich . Lassen Sie uns alle diese setzen Ψ ich in einen Vektor Ψ = ( Ψ 1 , Ψ 2 , ) , von dem nur vernünftigerweise gesagt werden kann, dass es darin lebt × ich H ich im Augenblick. In Anbetracht dessen die Cauchy-Folge Ψ 1 , Ψ 2 , soll in der Vollendung konvergieren v , und komponentenweise konvergiert es gegen ( Ψ 1 , Ψ 2 , ) = Ψ , können wir diese Cauchy-Sequenz natürlich mit dem Vektor identifizieren Ψ , die eine Funktion aus der Menge der Cauchy-Folgen in ist v Zu × ich H ich .

Es gibt einige Dinge zu überprüfen. Diese sind alle wichtig, um rigorose Mathematik zu betreiben, aber für physikalische Zwecke können Sie daraus schließen, dass sie nicht so wichtig sind. Die Intuition ist alles in der oben genannten Arbeit; Dies ist nur zusätzliche Arbeit, um sicherzustellen, dass das, was wir tun, Sinn macht, obwohl es auch eine anständige Möglichkeit ist, zu überprüfen und sicherzustellen, dass Sie verstehen, was vor sich geht. Wir müssen überprüfen, ob zwei Cauchy-Folgen in derselben Äquivalenzklasse an dieselbe gesendet werden Ψ . Dadurch wird sichergestellt, dass die Funktion von der Fertigstellung an als eine angesehen werden kann v Zu × ich H ich (oder dass es Faktoren durch die Fertigstellung von v in mathematischer Terminologie). Wir müssen auch überprüfen, ob es sich um eine lineare Karte handelt. Drittens müssen wir überprüfen, ob ein Element von v als eine konstante Cauchy-Folge angesehen wird, wird auf das gleiche Element von zurückgesendet v , so dass die natürliche Einbettung von v im Abschluss von v und in × ich H ich kompatibel sind. Und wir müssen zeigen, dass der Kern dieser linearen Abbildung trivial ist, sodass keine zwei verschiedenen Vektoren in der Vervollständigung von v sind auf dasselbe abgebildet in × ich H ich .

Nachdem all dies erledigt ist, ist es sinnvoll, den Abschluss von zu identifizieren v mit einer Teilmenge von × ich H ich als Vektorraum, da wir einen Vektorraum-Isomorphismus zwischen den beiden haben, der mit der Kopie von kompatibel ist v die beide enthalten. Jetzt können wir also sagen, dass die Fertigstellung von v ist (im Wesentlichen) ein Unterraum von × ich H ich . Es gibt mehr Struktur bei der Fertigstellung von v , nämlich dass es ein Hilbert-Raum ist, also ein inneres Produkt hat. Die letzte mathematische Sache, die Sie überprüfen müssen, ist, dass das innere Produkt am Ende eine schöne Form hat v In × ich H ich . Tatsächlich ist es nur Ψ | Ψ ' = ich = 1 Ψ ich | Ψ ich ' ich , wobei die unendliche Summe komplexer Zahlen wie üblich als Grenzwert von Partialsummen interpretiert wird. Diese Summe konvergiert garantiert (absolut) zu einem endlichen Wert für jedes Paar Ψ , Ψ ' dass wir auswählen, solange sie beide in der Fertigstellung liegen v In × ich H ich .

Nachdem all das aus dem Weg geräumt ist, kehren wir zum Aufrufen zurück ich H ich , eher als die Fertigstellung von v . Immerhin reden wir darüber, aber ich habe diese Sprache bisher vermieden, da wir noch nicht genau wussten, was sie bedeutet.


Nachdem das jetzt aus dem Weg geräumt ist, ist es nicht so schwer zu verstehen, warum Ψ = ( Ψ 1 , Ψ 2 , ) befriedigend ich Ψ ich ich 2 < ist eine notwendige und hinreichende Bedingung für Ψ angesagt sein ich H ich . Es ist notwendig, weil z Ψ ich H ich , wir brauchen Ψ | Ψ = ich = 1 Ψ ich 2 zu definieren, also ich = 1 Ψ ich 2 < erforderlich.

Der Vollständigkeit halber wollen wir in eine Cauchy-Folge konstruieren v konvergiert zu Ψ In ich H ich (Abschluss von v ), um zu zeigen, dass jedes Element, das diese Ungleichung erfüllt, in enthalten sein muss ich H ich . Das ist ziemlich einfach. Lassen Ψ 1 = ( Ψ 1 , 0 , 0 , 0 ) , Ψ 2 = ( Ψ 1 , Ψ 2 , 0 , 0 , ) , Ψ 3 = ( Ψ 1 , Ψ 2 , Ψ 3 , 0 , ) usw. Beachten Sie, dass all dies Ψ J sind in v da hat jeder nur J Elemente ungleich Null. Solange die Folge Cauchy ist, konvergiert sie eindeutig gegen Ψ . Die Aussage, dass Ψ 1 , Ψ 2 , Cauchy ist genau das Richtige für jeden ϵ > 0 , gibt es einen Index N so dass für jede Wahl von J , k > N , Ψ J Ψ k 2 < ϵ 2 (Ich habe beide Seiten der Einfachheit halber quadriert). Ohne Verlust der Allgemeinheit können wir nehmen J > k , in diesem Fall vereinfacht sich diese Aussage zu Ψ k + 1 k + 1 2 + + Ψ J J 2 > ϵ 2 . Nun ist die linke Seite immer kleiner als die unendliche Summe im Grenzwert J und wo k = N , also wollen wir finden N so dass l = N | Ψ | l 2 < ϵ 2 . Seit ϵ war positiv, ist es auch ϵ 2 . Es ist ein allgemeines Theorem in der fortgeschrittenen Analysis, dass, wenn eine Reihe nicht negativer Terme erfüllt sind l = 1 A l < , dann für jede positive Konstante (was wir annehmen werden ϵ 2 hier), gibt es einen Schwanz der Folge, dessen Summe kleiner als diese Konstante ist, dh es existiert ein N so dass l = N A l < ϵ 2 . Wenden Sie diese hier mit an A l = Ψ l l 2 , sehen wir, dass wir eine solche bekommen N , also ist dies eine Cauchy-Folge und Ψ ist in ich H ich und das Leben ist gut.


Sie fragen sich vielleicht, warum ich viele Absätze gebraucht habe, um zu erklären, was Wald in einem kurzen Absatz tut. Der Grund (glaube ich) ist, dass Wald implizit davon ausgeht, dass dies einigen Lesern aus der Funktionsanalyse bekannt sein wird, und es ist sowieso nicht wichtig genug, ihm einen Abschnitt zu widmen, da es von geringer physikalischer Relevanz ist.

3. Wie passt diese Definition der direkten Summe zu den üblichen Dingen, die wir sehen, wenn wir Tensoren in der allgemeinen Relativitätstheorie oder in Darstellungen von Lie-Algebren usw. betrachten?

Es passt überhaupt nicht zusammen. Direkte Summen haben überhaupt nichts mit Tensorprodukten zu tun. Sie sind viel enger mit direkten Produkten verwandt. Tatsächlich sind sie gleich, wenn das Produkt über einer endlichen Indexmenge liegt, aber für unendliche Indexmengen müssen wir beginnen, die Dinge umzustellen. Unendliche Tensorprodukte auf Hilbert-Räumen sind in der Regel noch hässlichere Dinge, die wir glücklicherweise normalerweise vermeiden können.

Sie haben wahrscheinlich Erfahrung mit direkten Summen/Produkten über endlichen Indexsätzen (wo sie gleich sind). Zum Beispiel lernen Sie in E&M und GR und anderen Kursen*, dass ein allgemeiner Tensor auf Rang 2 in einen skalaren Teil, der die Spur darstellt, einen antisymmetrischen Teil und einen symmetrischen spurlosen Teil zerlegt werden kann. In 4 Dimensionen zerlegt dies einen 16-dimensionalen Vektorraum in die direkte Summe (oder das Produkt) eines 1-dimensionalen Raums, eines 6-dimensionalen Raums und eines 9-dimensionalen Raums. Die Konstruktion, die Wald macht, setzt unendlich viele Räume zusammen, also ist es komplizierter.

Ein weiteres elementares Beispiel dafür ist die Untersuchung der Drehimpulsaddition. Dies ist ein Fall endlicher direkter Summen von Hilbert-Räumen, genau das, wonach wir suchen. Sie erinnern sich wahrscheinlich, dass beispielsweise das Tensorprodukt eines Spin-2-Systems mit einem Spin-1-System über Clebsch-Gordon-Koeffizienten in einen Spin-3-Anteil, einen Spin-2-Anteil und einen Spin-1-Anteil zerlegt werden kann . Wenn N ¯ bezeichnet den Hilbert-Raum für ein Spin-n-System, dies ist nur die Aussage, dass 2 ¯ 3 ¯ = 1 ¯ 2 ¯ 3 ¯ . Dies ist eine endliche direkte Summe, aber Sie können sich vorstellen, dass Sie für eine unendliche Anzahl von Teilchen eine unendliche direkte Summe erhalten könnten. Diese "unendliche Anzahl von Teilchen" ist nicht physikalisch (obwohl wir viele Modelle wie das Ising-Modell haben, die endliche Systeme zB in kondensierter Materie als unendlich annähern), aber es wird physikalisch, wenn Sie anfangen, an die "Teilchen" zu denken. als lokalisierte Erregungen, die an jedem Punkt im Raum sein können. Das ist wahrscheinlich der Grund, warum dies in Walds Buch auftaucht, obwohl ich es nicht genau weiß, da ich es nicht gelesen habe.

Die direkte Produktkonstruktion ist auch beim Studium der Lie-Algebren üblich, nach der Sie gefragt haben. Das direkte Produkt von Lie-Algebren G Und H ist nur ihr direktes Produkt als Vektorräume, denen wir diese Elemente auferlegen G pendeln mit denen von H . Es handelt sich also um Paare ( X , j ) G × H mit [ ( X 1 , j 1 ) , ( X 2 , j 2 ) ] = ( [ X 1 , X 2 ] , [ j 1 , j 2 ] ) . Sie könnten dieses Produkt auf unendlich viele Faktoren erweitern G ich wenn du wirklich wolltest. Wenn Sie jeweils eine Hilbert-Raumstruktur hätten G ich , könnten Sie auch ihre direkte Summe \bigoplus_i \mathfrak g_i$ wie oben definieren, was sowohl die Hilbert-Raumstruktur als auch die Lie-Algebra-Struktur erben würde, aber es wäre nicht das, was wir normalerweise meinen, wenn wir von unendlichen direkten Summen von Lie-Algebren sprechen (siehe nächster Absatz). Eine solche Konstruktion hat mit keiner Physik zu tun, die mir im Moment einfällt.

Eine der Schwierigkeiten bei der Suche nach einem einfachen Analogon besteht darin, dass es sich um Hilbert-Räume handeln muss. Die direkte Summe in der Kategorie der Hilbert-Räume ist völlig anders als in der Kategorie der Vektorräume. Als Vektorräume (unter Vergessen der Hilbertraumstruktur) ich H ich = v , nicht der Abschluss von v . Alles in #2 über analytisches Zeug hört einfach auf, relevant zu sein. Es ist also sehr wahrscheinlich, dass Sie zuvor keine unendlichen direkten Summen von Hilbert-Räumen benötigt haben, da sie in der QFT auf flacher Raumzeit sowie in der gewöhnlichen QM fast immer vermieden werden können.


Ich würde auch nicht vorschlagen, sich zu sehr damit zu beschäftigen. In der Physik versuchen wir normalerweise, unsere direkten Summen endlich zu halten. Wenn sie unendlich werden, neigen wir dazu, die analytischen Schwierigkeiten so weit wie möglich zu ignorieren und die vollständig strenge Analyse den Mathematikern zu überlassen. Ich vermute, dass es viele praktizierende theoretische Physiker gibt, die Ihnen keine strenge Definition für die unendliche direkte Summe geben könnten, ohne sich vorher etwas anzusehen. Nicht, dass es besonders schwer wäre, aber die feinen Unterschiede zwischen unendlichen Summen und Produkten von Hilbert-Räumen spielen für die praktische theoretische Physik keine große Rolle. Viele Berechnungen würden angestellt v oder hinein × ich H ich eher als drin ich H ich . Sie gehen normalerweise gut aus, außer in seltenen Fällen, in denen die Antworten eindeutig unsinnig sind. Es sollte betont werden, dass die Konstruktion der direkten Summe buchstäblich die einzig mögliche vernünftige Wahl für eine direkte Summe von Hilbert-Räumen ist. Wenn Sie also befürchten, dass die analytischen Unterschiede zu einigen physikalisch relevanten Problemen führen werden, hängt davon ab, ob Sie sich entschieden haben auf die eine oder andere Weise, es in einem philosophischen Sinne zu definieren, kann das nicht passieren, vorausgesetzt, das, was Sie tun, ist überhaupt konsistent.

Das bedeutet nicht, dass ich denke, dass Sie es ignorieren sollten, aber lassen Sie sich davon nicht verwirren. Wenn Sie eine unendliche direkte Summe von Hilbert-Räumen sehen, stellen Sie sich das einfach als unendliche Version einer endlichen direkten Summe vor, bis Sie es sich absolut nicht mehr leisten können. Um strenge mathematische Physik zu betreiben, brauchen Sie die Unterscheidung, aber für die meisten theoretischen Physik müssen Sie nicht zu vorsichtig damit sein. Wald ist ziemlich mathematisch, was nicht unbedingt schlecht sein muss, aber man sollte die Mathematik nicht mit der Physik verwechseln, und hier gibt es keinen physikalischen Inhalt.

*In Wirklichkeit sind einige der obigen Ausführungen zu Tensoren in GR, E&M usw. eine bequeme Lüge. Wir interessieren uns normalerweise für Tensorfelder (die Physiker normalerweise nur Tensoren nennen), die (normalerweise glatte) Abschnitte eines Bündels auf einer Raumzeit-Mannigfaltigkeit sind. Die Tensorfelder selbst sind also Elemente unendlichdimensionaler Vektorräume. Normalerweise berufen wir uns auf eine Art Lokalitätsprinzip, so dass wir uns nur an einem Punkt mit Tensoren befassen müssen, die dann einen endlichdimensionalen Raum bilden. Der Raum der Abschnitte ist nicht die direkte Summe der einzelnen Räume, sondern eine andere Art, sie zusammenzusetzen, die von Natur aus intuitiv additiv ist. Es ist jedoch kein Hilbert-Raum, auch wenn jeder der einzelnen Räume ein Hilbert-Raum ist. Nur wenn man davon ausgeht, dass die Raumzeit-Mannigfaltigkeit kompakt ist, oder einige analytische Bedingungen an die Arten von Abschnitten stellen, die wir zulassen, können Sie eine endlichwertige Norm erhalten, indem Sie die inneren Produkte über die gesamte Mannigfaltigkeit integrieren. Dies ist ein alternativer Ansatz für globale Konstruktionen als der obige, eher im Geschmack der klassischen Feldtheorie.

@ Noix07 Ich sehe kein Problem darin, lange Antworten zu schreiben. Eine andere Sichtweise, auch wenn sie sehr ähnlich zu dem ist, was andere geschrieben haben, kann für jemanden wertvoll sein. Wenn Sie das alles nicht lesen wollen, ist das in Ordnung, aber so zu kommentieren, wie Sie es getan haben, ist meiner Meinung nach nicht hilfreich.
Dies ist eine großartige Antwort, vielen Dank! Es verdient viel mehr Upvotes, imo.

Ein Element der direkten Summe H 1 H 2 . . . ist eine Folge

Ψ = { Ψ 1 , Ψ 2 , . . . }
bestehend aus einem Element aus H 1 , und Element aus H 2 ...etc (zählbar). Diese muss die besondere Eigenschaft haben, dass die Summe
Ψ 1 2 + Ψ 2 2 + . . .
konvergiert. Diese Konvergenz ist Teil der Definition der direkten Summe.

Um zu zeigen, dass die direkte Summe ein Hilbert-Raum ist, brauche ich eine wohldefinierte Additionsoperation. Wenn ich so ein Paar habe:

Ψ = { Ψ 1 , Ψ 2 , . . . }
Φ = { Φ 1 , Φ 2 , . . . }
Ich kann ihre Summe als Folge definieren
Φ + Ψ = { Φ 1 + Ψ 1 , Φ 2 + Ψ 2 , . . . }
Das müssen wir überprüfen
Ψ 1 + Φ 1 2 + Ψ 2 + Φ 2 2 + . . .
konvergiert. Dazu nutzen wir die Tatsache, dass die einzelnen Begriffe genügen
Ψ ich + Φ ich 2
= Ψ ich 2 + Φ ich 2 + ( Ψ ich , Φ ich ) + ( Φ ich , Ψ ich )
Ψ ich 2 + Φ ich 2 + 2 Ψ ich Φ ich
2 Ψ ich 2 + 2 Φ ich 2
Konvergenz folgt also aus der Summierung der Konvergenzeigenschaften der einzelnen Räume. Das zeigt also, wie wir Elemente der direkten Summe hinzufügen können. Skalarprodukte folgen auf die gleiche Weise.

Um ein inneres Produkt zu definieren, addieren wir einfach die inneren Produkte komponentenweise, dh

( Ψ , Φ ) = ( Ψ 1 , Φ 1 ) + ( Ψ 2 , Φ 2 ) + . . .       ( 1 )
Um zu zeigen, dass die RHS konvergiert, beachten Sie
| ( Ψ ich , Φ ich ) | Ψ ich Φ ich
Aber
2 Ψ ich Φ ich Ψ ich 2 + Φ ich 2
also konvergiert die RHS von (1) absolut und wir sind fertig – wir haben einen wohldefinierten inneren Produktraum.

Ist es erforderlich, dass das Skalarprodukt konvergiert?
Es ist nur so, dass wir, wenn wir das Skalarprodukt durch Gleichung (1) definieren, eine endliche Antwort erhalten müssen, also muss die Summe auf der rechten Seite konvergieren. Wenn dies nicht der Fall wäre, wäre die innere Produktdefinition nicht sinnvoll.
@Prahar Das innere Produkt muss eine positive reelle Zahl sein. ist keine reelle Zahl, und das Skalarprodukt darf nirgendwo undefiniert sein.

Hinweise:

  1. Lassen Sie die Elemente von × a A H a heißen

    (1) Ψ   :=   ( Ψ a ) a A ,
    und so weiter.

  2. Nächste Runde × a A H a in einen normierten Raum a la Pythagoras:

    (2) | | Ψ | |   :=   a A | | Ψ a | | a 2
    (mit möglichen unendlichen Normvektoren erlaubt).

  3. Als nächstes definieren Sie den Direktsummenraum

    (3) a A H a   :=   { Ψ × a A H a | | Ψ | | < } .

  4. Leite das ab a A H a wird zu einem Vektorraum unter den Zuweisungen

    (4) λ Ψ   :=   ( λ Ψ a ) a A ,
    (5) Ψ + Φ   :=   ( Ψ a + Φ a ) a A .

  5. Lassen Ψ , Φ a A H a .

  6. Leiten Sie das aus der Cauchy-Schwarz-Ungleichung ab

    (6) a A | | Ψ a | | a | | Φ a | | a < .

  7. Inneres Produkt definieren

    (7) Ψ | Φ   :=   a A Ψ a | Φ a a .
    Sie ist wohldefiniert, da die Summe auf der rechten Seite absolut konvergent ist, vgl. Gl. (6).

  8. Fahren Sie außerdem fort, Sesquilinearität, Vollständigkeit usw. zu zeigen.

  9. Lassen Sie uns das schließlich erwähnen, wenn überhaupt a A : { e ich a | ich ICH a } ist eine Grundlage für H a , Dann { e ich a | ich ICH a , a A } ist eine Grundlage für a A H a .